LSAT and Law School Admissions Forum

Get expert LSAT preparation and law school admissions advice from PowerScore Test Preparation.

User avatar
 Dave Killoran
PowerScore Staff
  • PowerScore Staff
  • Posts: 5852
  • Joined: Mar 25, 2011
|
#85997
Complete Question Explanation
(The complete setup for this game can be found here: lsat/viewtopic.php?f=303&t=7341)

The correct answer choice is (E)

The new condition in the question stem results in the following sequence:
Capture27.PNG
Because K still cannot attend the first class, this sequence forces a number of students into certain classes. Here is the scenario, with distribution information:
Capture28.PNG
Only H, N, and S are not accounted for on the above diagram. Because either N or S must still attend the first class (remember, H is a female and from the third rule cannot attend the first class), we can infer that H and the N/S remainder attend the sixth class. Accordingly, answer choice (E) is correct.
You do not have the required permissions to view the files attached to this post.
 SherryZ
  • Posts: 124
  • Joined: Oct 06, 2013
|
#12869
Oct 1999 LSAT, Sec 3 Game #4, Q24:

Why does H must attend 6th? I thought she can attend 5th too, as following:

K-IL-O-G-PHN-S,
or
K-O-IL-G-PHS-N.


Could you help me out? Thank you so much for your help!

---Sherry
User avatar
 KelseyWoods
PowerScore Staff
  • PowerScore Staff
  • Posts: 1079
  • Joined: Jun 26, 2013
|
#12874
Hi Sherry!

K can't be first because the 3rd rule says that she is not the first student. So if O and IL are between K and G now, that forces K into the 2nd slot, O and IL in 3/4 in some order, G into 5, and PH into 6:

N/S -- K -- O/IL -- IL/O -- G -- P H S/N

Hope that helps!

Best,
Kelsey

Get the most out of your LSAT Prep Plus subscription.

Analyze and track your performance with our Testing and Analytics Package.